12.07.2015 Views

Cálculo Matricial

Cálculo Matricial

Cálculo Matricial

SHOW MORE
SHOW LESS

Create successful ePaper yourself

Turn your PDF publications into a flip-book with our unique Google optimized e-Paper software.

inferior ⎡ com 1’s na ⎤ diagonal e U matriz escada para as quais PA = LU. ⎡ Para tal, basta ⎤ tomar0 1 0−1 3 0⎢ ⎥P = ⎣ 1 0 0 ⎦, L = (E 32 (−3)E 31 (2)) −1 ⎢ ⎥= E 31 (−2)E 32 (3), e U = ⎣ 0 3 −2 ⎦.0 0 10 0 1Considere agora a matriz M =⎡⎢⎣1 1 10 0 11 0 1⎤⎥⎦. Ora E 31 (−1)M =força a troca da segunda pela terceira linha. Obtemos, assim, P 23 E 31 (−1)M =⎡⎢⎣1 1 10 0 10 −1 0⎡⎢⎣⎤⎥⎦, o que1 1 10 −1 00 0 1que é uma matriz escada. Neste caso, como se obtêm as matrizes P,L,U do teorema? Aocontrário do exemplo anterior, a realização matricial das operações elementares por linhas doAEG não nos fornece, de forma imediata, essa factorização. No entanto, poder-se-ia escrever⎡⎢E 31 (−1)M = P 23 ⎣1 1 10 −1 00 0 1⎤⎥⎦, já que P −1⎢⎣23 = P 23, e portanto M = E 31 (1)P 23⎡1 1 10 −1 00 0 1pois E 31 (−1) −1 = E 31 (1). Note que E 31 (1)P 23 ⎡ ≠ P 23 E 31 (1). ⎤ Não obstante, repare que1 1 1⎢ ⎥E 31 (1)P 23 = P 23 E 21 (1), donde M = P 23 E 21 (1) ⎣ 0 −1 0 ⎦, e portanto PA = LU, com0 0 1P = P 23 ,L = E 21 (1) e U =⎡⎢⎣1 1 10 −1 00 0 1⎤⎥⎦.Lema 3.3. Para i,k,l > j, e para todo o a ∈ K, é válida a igualdade E ij (a)P kl = P kl E lj (a).Demonstração. Se k ≠ i, então a igualdade é óbvia.Suponha que k = i. Pretende-se mostrar que E ij (a)P il = P il E lj (a), com i,l > j. SendoP il E lj (a) a matriz obtida de E lj (A) trocando as linhas i e l, e visto a linha l de E lj (a) serentão a linha i de P il E lj (a) é[0 · · · 0 a 0 · · · 0 1 0 · · · 0]↑↑j l⎤⎥⎦,⎤⎥⎦,[0 · · · 0 a 0 · · · 0 1 0 · · · 0]↑↑j l.E ij (a)P il é a matriz obtida de P il a que à linha i se somou a linha j de P il multiplicadapor a. Sendo a linha i de P il[0 · · · 0 · · · 0 1 0 · · · 0]31↑l

Hooray! Your file is uploaded and ready to be published.

Saved successfully!

Ooh no, something went wrong!